K
Khách

Hãy nhập câu hỏi của bạn vào đây, nếu là tài khoản VIP, bạn sẽ được ưu tiên trả lời.

18 tháng 1 2017

Theo bài ra , ta có : 

\(A=\frac{x}{\left(x+2017\right)^2}\left(x>0\right)\)

\(\Rightarrow A=\frac{x}{x^2+4034x+2017^2}\)

Vì x>0 nên ta chia cả tử và mẫu của biễu thức A cho x \(\left(x\ne0\right)\)

Suy ra \(\frac{1}{x+4032+\frac{2017^2}{x}}\) 

Vì \(\left(x>0\right)\Rightarrow\frac{2017^2}{x}>0\)

Áp dụng BĐT Côsi với 2 số x>0 và \(\frac{2017^2}{x}>0\)

Ta có : \(x+\frac{2017^2}{x}\ge2\sqrt{x+\frac{2017^2}{x}}\)

\(\Rightarrow x+\frac{2017^2}{x}\ge2.2017\)

\(\Rightarrow x+\frac{2017^2}{x}\ge4034\)

\(\Rightarrow x+\frac{2017^2}{x}+4034\ge8068\)

\(\Rightarrow\frac{1}{x+\frac{2016^2}{x}+4032}\le\frac{1}{8064}\Rightarrow A\le\frac{1}{8064}\)

\(\Rightarrow MaxA=\frac{1}{8064}\)

Dấu bằng xảy ra \(\Leftrightarrow x=\frac{2017^2}{x}\) 

                         \(\Leftrightarrow x^2=2017^2\) 

                      Vì x > 0 

                         \(\Rightarrow x=2017\)

Vậy MaxA = \(\frac{1}{8064}\)khi và chỉ khi x = 2017 

Chúc bạn học tốt =)) 

4 tháng 2 2017

Hỏi @ CTV

Nếu không có đk x>0

thì liệu A có GTLN không?  

19 tháng 1 2017

Ta có: \(a^2+b^2+c^2=\left(a+b+c\right)^2\)

\(\Leftrightarrow ab+bc+ca=0\)

Ta có: \(A=\frac{1}{a^2+2bc}+\frac{1}{b^2+2ac}+\frac{1}{c^2+2ab}\)

\(=\frac{1}{a^2+2bc-ab-bc-ca}+\frac{1}{b^2+2ca-ab-bc-ca}+\frac{1}{c^2+2ab-ab-bc-ca}\)

\(=\frac{1}{a^2+bc-ca-ab}+\frac{1}{b^2+ca-ab-bc}+\frac{1}{c^2+ab-bc-ca}\)

\(=-\left(\frac{1}{\left(a-b\right)\left(c-a\right)}+\frac{1}{\left(b-c\right)\left(a-b\right)}+\frac{1}{\left(c-a\right)\left(b-c\right)}\right)\)

\(=-\frac{b-c+c-a+a-b+}{\left(a-b\right)\left(b-c\right)\left(c-a\right)}=0\)

PS: Hồi tối lười để người khác làm mà không ai làm thôi t làm vậy

18 tháng 1 2017

( a+b+c)^2 = a^2 + b^2 + c^2 

=> a^2 + b^2 + c^2 + 2ab + 2bc + 2ac = a^2 + b^2 + c^2 

=> a^2 + b^2 + c^2 + 2ab + 2bc + 2ac - a^2 - b^2 - c^2 = 0 

=> 2ab + 2bc + 2ac = 0 

ta có 

A = \(\frac{1}{a^2+2bc}\)\(\frac{1}{b^2+2ac}\)\(\frac{1}{c^2+2ab}\)

=  \(\frac{1}{a^2+2bc}\)\(\frac{1}{b^2+2ac}\)\(\frac{1}{c^2+2ab}\) + 2ab + 2bc + 2ac 

đến đây bạn nhóm lại nhé mk giải ra thì dài lắm nên chỉ gợi ý cho bn đấy đây thôi

18 tháng 1 2017

ko biết

18 tháng 1 2017

câu này thi bn quy đòng bình thường mà tính thôi 

khai triển ra 

rồi tạo ra x= y để thay vào bạn cứ biến đổi 

như vậy thì sẽ ra thôi

18 tháng 1 2017

Mình thích nhân phân phối không thích ghép

(x-3)^3=x^3-3.3.x^2+3.3^2.x-3^3

ok

(x-3)^3=27+19=54

\(x=3+\sqrt[3]{54}=3+3\sqrt{2}\)

18 tháng 1 2017

Bài này giải theo phương trình tích 

  Ta có :  x^3 - 9x^2 + 27x =19

         <=> x^3 - 9x^2 + 27x -19 = 0

         <=>  x^3 - x^2 - 8x^2 + 8x + 19x -19 = 0

         <=>  x^2(x-1) - 8x(x-1) + 19(x-1) = 0

         <=>  (x-1)(x^2 - 8x + 19) = 0

  Ta CM được x^2 - 8x + 19 >0  

       => x-1= 0  <=> x=1

  Vậy phương trình có nghiêm x=1

18 tháng 1 2017

Bài 2 thay 2 vào x rồi giải bình thường tìm k 

18 tháng 1 2017

Nhìn sơ qua thì thấy bài 3, b thay -2 vào x rồi giải bình thường tìm m

18 tháng 1 2017

Bài 2:

a) \(x+x^2=0\)

\(\Leftrightarrow x\left(x+1\right)=0\)

\(\Leftrightarrow\hept{\begin{cases}x=0\\x+1=0\end{cases}}\)\(\Leftrightarrow\hept{\begin{cases}x=0\\x=0-1\end{cases}}\)\(\Leftrightarrow\hept{\begin{cases}x=0\\x=-1\end{cases}}\)

b) \(0x-3=0\)

\(\Leftrightarrow0x=3\)

\(\Rightarrow vonghiem\)

c) \(3y=0\)

\(\Leftrightarrow y=0\)